ASWB

Réussis tes devoirs et examens dès maintenant avec Quizwiz!

A social worker meets with an immigrant couple because of increasing frustration and fighting. Before the couple immigrated a year ago, the husband worked and his wife stayed home. Since moving, the husband has had difficulty finding work due to limited English skills. The wife speaks English fluently and has been able to find full-time work. The wife describes her husband as increasingly irritable and always wants her to stay home to take care of the house. The social worker should FIRST consider: A. Address how the change in roles is affecting their relationship. B. Explore the reasons for the couples immigration. C. Referring the couple to a group for immigrant families. D. Referring the husband for vocational counseling.

A starts where the clients are and addresses their current situation. B, C, and D could be later options.

Social work supervisors engaging in activities that help the supervisee to: increase self-awareness, refine knowledge and skills, develop professionally as it pertains to their specific role, deal with ethical issues that arise,as well as learn appropriate techniques around assessment, intervention, and treatment planning is engaging in what type of supervision activities? A. Administrative B. Educational C. Supportive D. Clinical

B. These fall into the educational component of supervision described by NASW and ASWB's document on the Best Practice Standards in Social Work Supervision: www.socialworkers.org/LinkClick.aspx?fileticket=GBrLbl4BuwI%3D&portalid=0

All of the following may be part of the social planning process EXCEPT: A. Holding focus groups B. Engaging in policy activism C. Facilitating town meetings D. Hosting community forums

B. This is a recall question. A, C, and D are all part of the planning process. Engaging in policy activism (B) is action oriented and comes after the planning process.

A school counselor refers a family to a social worker after finding their 7-year- old child pretending to have sexual intercourse with another child at recess. The mother states, "I'm so embarrassed about this happening at school, and not sure what to do." What should the social worker do FIRST: A. Report suspected child abuse to the proper authority. B. Provide psychoeducation on normal child development. C. Assess for further signs for sexual abuse. D. Consult with the school counselor to obtain further details about the incident.

C This is outside of normal behavior for a 7-year-old, so B is out. Ruling our sexual abuse would be the first step. A is premature and D could be done after talking with the parents.

A social worker is part of an evaluation team assessing the effectiveness of a neighborhood outreach program targeting families with children ages 0-5. The social worker is in charge of collecting qualitative data. Which of the following is the BEST example of a qualitative measure? A. Looking at the way variables such as age and income level impact the utilization of client services. B. Tracking the number of clients served in a year through a specific program C. Comparing the number of clients using a new service compared to an old service D. Interviewing clients about their level of satisfaction with the changes in program delivery

D is the best answer because it involves gathering information directly from program participants. A, B, and C are all examples of quantitative data.

A social worker is working with a client who recently came out as transgender to his parents. The client has expressed fear around transitioning because he is afraid of disappointing his family. The social worker is transgender and feels strongly about the client owning his transition, but is also aware that this countertransference may affect the development of a treatment plan. Which of the following is the MOST important component to keep in mind while developing a treatment plan? A. Ensuring the client is aware of the countertransference B. Keeping the family's perspective in mind C. Making sure an outside supervisor has consulted on the treatment plan D. Ensuring the professional code of ethics is followed

It's most important to be aware of the professional code of ethics, as this will allow the social worker to remain impartial as the treatment plan is developed. A would not be beneficial for the client in terms of developing his treatment plan and would be inappropriate. B would undermine the client's wishes. There is nothing in the stem to warrant C.

A woman seeks therapy 2 months after miscarrying for the second time. The client reports recurring dreams about losing the baby and feelings of anger when she sees women with healthy babies in public. Her husband is refusing to attend therapy, saying that "everything happens for a reason." The social worker's primary task is to: A. Provide psychoeducation on the grieving process. B. Explore the client's sadness. C. Support the client through her grieving. D. Explore the client's relationship with her husband.

Start where the client is. A may be helpful, but is not the primary task. D is related to the husband who is not in the room. B is not indicated. C is the only answer that addresses the client's presenting problem.

A social worker is approached by a mother regarding her 2 ½ year-old daughter. The mother is distressed by the fact that her daughter frequently says "NO" when asked to clean up after herself, has occasional temper tantrums and wants to do "everything" independently. The social worker's BEST response would be to: A. Educate the mother about child development. B. Schedule a biopsychosocial evaluation of the child. C. Review possible behavior modification techniques. D. Engage the mother and daughter in family therapy.

The child isexhibiting normal, healthy behavior for a 2-year-old. A would be the best placeto start.

A 15-year-old is emancipated from her parents. Each of the following is true of emancipation for a 15-year-old EXCEPT? A. They are legally considered an adult and can therefore choose to drop out of school B. They are legally declared an adult by a judge and make decisions about their own health and welfare. C. They are no longer under the care or control of their parent/legal guardian D. They can apply for public assistance

The correct answer is A. This is a recall question. A is not true of emancipated clients.

Assertiveness Therapy is an Evidence-Based therapy characterized by all of the following EXCEPT: A. It helps the client engage in actions that reflect their own best interest B. It teaches the client to stand up for oneself without experiencing significant anxiety C. It helps the client express their feelings comfortably D. It assists the client in exercising their own rights and denying the rights of others

The correct answer is D. D is not a part of Assertiveness Therapy. Assertiveness Therapy helps the client exercise their own rights, but this is NOT done in a way that denies the rights of others.

A social worker is working with a family with three children, the oldest of whom, age 14, is in a wheelchair because of cerebral palsy. The parents describe the child's lack of interest in family activities and believe that he is depressed. The child remarks that his mother hovers over him, and they never let him go anywhere alone. He expresses an interest in going to special needs summer camp, but his parents shake their heads. What should the social worker do FIRST? A. Support the parents' understandable need to be protective. B. Refer the 14-year-old for a medication evaluation. C. Provide psychoeducation about normal adolescent development. D. Encourage parents to support their son's independence.

The parents areconcerned about the 14-year-old's behavior, but his behavior sounds healthy forhis age. Of the answers given, C would be the best place to start.

All of the following are common traits of child sexual abuse perpetrators EXCEPT: A. They are generally someone that is known and trusted by the child. B. They have a history of abuse themselves. C. They are usually strangers and unknown to the child. D. They are generally male.

This is a recall question regarding knowledge about common traits of child sexual abuse perpetrators. C is the best answer because children generally know their perpetrator (such as a relative, family friend, or trusted adult).

A social worker in a small community is contacted by a prospective client regarding therapy for conflicts she is experiencing at her job. The social worker knows that the client works at the agency where the social worker was previously a supervisor, although the social worker did not supervise the client. The prospective client insists that the social worker's knowledge of the agency environment and culture will benefit treatment because of increased understanding. What is the social worker's MOST appropriate response? A. Decline to provide therapy due to knowledge of the previous business relationship. B. Assure the client of confidentiality before beginning treatment. C. Provide treatment while maintaining self-awareness regarding boundary issues. D. Explore the meaning of dual relationships before beginning treatment.

A and D are yourbest options for this questions. A is a little stronger, because, in this case,it would be better to refer her to someone not connected to the previousworking environment. Providing a safe space for clients is an essential part oftherapy. A client like this may assumethat you will agree with her view of the situation, but your previous knowledgeof the environment could actually be a hindrance to your work together.

A social worker is hired by a non-profit foundation to complete a process evaluation on three agencies that received grants from the foundation. Each of the following is an example of a process evaluation question EXCEPT: A. Was the program more successful with certain groups of people than with others? B. Was the program well managed? C. Was there adequate agency support for the program? D. What specific interventions were put into place by the program?

A is the best answer because it is not a question associated with process evaluation; it is associated with program evaluation.

A new client comes to treatment after losing his job of 15 years. He reports that "nothing in his life is going right," and goes on to note that his marriage is failing, recent investments are not paying off, and that he has "no real friends." In addition to a risk assessment, the social worker should focus on: A. The client's strengths and resources B. A possible medical issue C. Referring the client for employment assistance D. A referral for couples therapy

A is the best answer, because it focuses on the client's strengths. B is not indicated by the client's symptoms or presenting problem. C and D may be helpful, but the social worker should identify the client's strengths and resources so that interventions and treatment decisions can be made appropriately.

A school social worker is meeting with a 17-year-old Latino student as part of her post-graduation assessment/planning. During the conversation, the student expresses a desire to go to college and then begins weeping and explains that as an undocumented student, most colleges will be out of reach. The social worker should FIRST: A. Assist the student in identifying colleges that will accept undocumented students B. Explain that the social worker cannot help since the student is technically breaking the law C. Refer the student to an immigration attorney D. Help the student process her feelings about being undocumented

A is the best answer, because it meets the student where she is and will help empower her and help her be hopeful about her future. B is in opposition with social work values. C may be needed at some point, but is not indicated at this time. D would be a longer term goal but would not be what the social worker does FIRST, particularly in a school setting.

A white social worker at a hospital is asked to evaluate a black woman who was recently brought in by the paramedics. When the social worker approaches the client, she refuses to tell the social worker what happened and states repeatedly that she fears anything she says to the social worker will be used against her. The woman requests to instead speak with the Latina nurse who has been caring for her since arrival. What is the MOST likely explanation for the woman's behavior? A. Cultural paranoia, which represents a healthy reaction to racism. B. An episode of acting out of Borderline Personality dynamics. C. Pathological paranoia regarding all social workers. D. Resistance to assessment and treatment.

A is the best answer, because it takes into account the cultural issues presented in the case. B and C both pathologize the client's behavior; D makes a negative judgment about the client's behavior, when it may in fact be protective.

A social worker meets with a veteran who has been diagnosed with PTSD. During a session, the client's eyes glaze over while he is talking about an experience in which several members of his platoon were killed. He sits forward in the chair, staring over the social worker's shoulder, and talks in a calm, monotonous voice. The social worker should: A. Employ a grounding technique to reconnect the client with the present moment B. Allow the flashback to continue so the client can express his feelings C. Assess the client for suicidal ideation D. Empathize with the client's feelings of fear and loss of control

A is the best answer, because the client is experiencing a flashback and the social worker should try to help him distinguish between what is happening now versus what happened in the past to decrease his distress. B is not a good answer, because therapeutically it is not helpful to allow flashbacks to continue. C is not indicated in this moment. D does not make sense since the client in the question is not expressing these feelings.

A social worker receives a referral for family therapy. The family consists of mother, father, a 14-year-old girl, and an 8-year-old boy. The parents report they want the family to spend "quality time" together, but whenever they try, everyone ends of up fighting. The daughter rolls her eyes and reports that her parents try to talk to her "like they are my friends." The 8-year-old says he wants to play with kids in the neighborhood and go to sleepovers, but the parents refuse, saying he can have "all of that fun right here at home." The social worker should focus treatment on: A. Helping the family establish appropriate roles and boundaries B. Assisting the family in healthy communication strategies C. Facilitating activity-based family interactions to build on relational strengths D. Creating connection through conflict resolution

A is the best answer, because the question stem indicates that the parents are trying to interact with their children like peers instead of parents. Helping the parents establish more appropriate roles with the children may decrease the tension and allow for more positive family interactions. B and D may be helpful but would not be the focus of treatment in this case. C will not be as helpful until the parents develop more appropriate boundaries and roles for themselves.

A 55-year-old female client is in therapy with a male social worker for help with her relationships with her grown sons. The social worker perceives that a good working relationship has been established, but in the eighth session, the client announces that she wants a new social worker. She states that the social worker is too young and inexperienced to help with her particular problem. The social worker should FIRST: A. Encourage the client to discuss her feelings about their relationship. B. Review his professional experience to reassure the client. C. Help the client to differentiate the social worker from her grown sons. D. Offer a list of older social workers and facilitate a referral.

A is the best answer, because the social worker should be discussing the client's feelings about the relationship. This would be the place to begin, so A is the best answer. B, C, and D could be done once the issue had been explored more.

A newly licensed social worker is providing outreach to a community that recently experienced a major hurricane. The social worker goes door-to-door, asking families if they need mental health services and outlining opportunities to receive these services. At the end of her first week, the social worker realizes that no families have expressed interest in receiving mental health services. The social worker should next: A. Attend upcoming community events to join with the community and understand their needs. B. Provide informational flyers in community centers to reach people in a less threatening way. C. Terminate the intervention since no one is interested. D. Continue the intervention as planned.

A is the best answer. The social worker should recognize that a community that has recently experienced a stressful or traumatic event may not be open to new people or offerings. Joining with the community and understanding their needs will assist the social worker in meeting those needs effectively. B might help but does not do as much to create a relationship between the social worker and the community. The social worker will not know unless s/he gets to know the community better. C is premature, and D continues with an intervention that is not working at this time.

A social worker wants to start a sexually transmitted infection primary prevention program for residents in a retirement and assisted living community. The social worker should FIRST: A. Meet with the residents to engage them in the process of designing the program. B. Have informational meetings on safe sex with residents. C. Screen all patients for STI's. D. Develop informational pamphlets on the risks of unsafe sex.

A is the best place to start. It is important to find out what the community needs and engage them in the planning process. B, C, and D could all be part of the program depending on the needs.

After eight sessions with a teenage girl who suffers from anxiety and depression, her mother calls and informs the social worker that they won't be bringing the daughter for therapy because they don't think it's helping. The social worker believes that the daughter is benefiting from therapy. The social worker should NEXT: A. Explore the mother's views that therapy isn't working. B. Provide referral to another social worker. C. Encourage the mother to allow the daughter to come for a termination session. D. Discuss the ways the social worker believes the client has improved.

A is the bestplace to start because the therapist can't appropriately move forward with anyother action before having some understanding of the mother's perspective. B, C, and D might all be appropriate, but notuntil the social worker understands why the mother doesn't think the treatmentis working.

A group of non-profit agencies are concerned about recent violence in the neighborhoods they serve. During a conference call, one of the program managers suggests forming a coalition. Which of the following statements is MOST true about coalitions?: A. They draw together representatives of a number of groups who have common interests or concerns regarding a particular issue B. They are made up of individuals in disagreement about a particular policy C. The effectiveness of a coalition is determined by the number of individuals in that coalition D. Members of coalitions come from similar backgrounds, allowing them to be in agreement on all issues surrounding a particular policy issue.

A is the correctanswer because it addresses the purpose of coalitions. B, C, and D are not accurate.

A social worker assesses an adult client during readmission to an inpatient psychiatric unit. The client is floridly manic, talking rapidly and loudly about his recent trip to Las Vegas, his most recent business venture and several other topics without pause. To complete a psychosocial assessment the social worker should FIRST: A. Look up previous hospital records to obtain information until the client is able to accurately respond to questions. B. Continue asking questions until the client responds appropriately. C. Contact the client's family and friends for information regarding his health. D. Contact previous treatment providers.

A. The client is being readmitted, which means there is already a chart. This would be the place to start. C and D would break confidentiality. He is not in the condition for B.

A social worker is treating a client who recently came out at work.The client was recently let go from employment due to "personnel conflicts." What should the social worker encourage the client to do? A. Meet with a lawyer regarding the termination. B. Apply for unemployment. C. Find a job at a company that is more accepting. D. Address the termination with the supervisor.

A. The client is upset and it appears to be possible discrimination. A is an option to consider and it would empower the client.

A client is being treated by a social worker. She has a history of relationship problems and suicidal gestures. In the first session, the client tells the social worker how great she thinks the social worker is. What is the MOST important element of a treatment plan for this client? A. Establishment of clear therapeutic boundaries with the client. B. Short-term cognitive therapy to work with distorted self-image. C. Long-term psychoanalytic therapy to process painful memories of the past. D. Adjunctive group treatment to address relationship problems.

A, because the client is presenting with several symptoms indicative of Borderline Personality Disorder (relationship problems, suicidal gestures, idealization of the social worker) and boundaries are a key component to engaging in effective treatment with a client with this profile. B, C, and D are not specifically related to treatment with clients with Borderline Personality Disorder.

A 55 year-old Native American woman seeks treatment following the death of her son. When working with Native American clients, in order to be culturally sensitive and develop a good therapeutic relationship it is important to observe all of the following EXCEPT: A. Be directive B. Use active listening and reflective responses. C. Avoid interrupting D. Allow for periods of silence

A. Answers B, C, and D are all advised approaches with Native Americans. A is the only one that should be avoided.

A social worker gathers a group of individuals from a variety of backgrounds and professions who share an interest in increasing the accessibility of affordable healthcare for underemployed and/or uninsured individuals. What approach is the SW using? A. Coalition Building B. Client Empowerment C. Social Networking D. Social Justice

A. Because the social worker is bringing together people from various backgrounds and professions for a single purpose, this can best be described as coalition building.

A social worker meets with a couple who are both in their 60's. The couple reports that they are constantly getting in arguments and finding it difficult to agree with one another. Their relationship had been good until recently and the couple isn't sure what has changed. The husband is preparing to retire after working for the same company for 25 years and has been scaling back his hours and spending more time at home. The social worker should FIRST: A. Explore the meaning of the husband's retirement and the impact it has on their relationship. B. Assign the couple homework of communication exercises. C. Address any trust issues between the husband and wife. D. Recommend individual therapy for the husband.

A. Changing roles can have an impact on relationships. A would be the place to start because of the recent change. B could be done later. C and D are not indicated at this time.

An emergency room social worker is asked to evaluate an 18 year-old pregnant female who was admitted after an accident at her job. The woman reports to the social worker that she hasn't seen a doctor "in a year or so," gets most of her meals at the fast-food restaurant where she works, gets very little sleep and enjoys "partying" with her friends on the weekends. The social worker should FIRST: A. Assess the client's knowledge of the effects of her behavior on the fetus. B. Refer the client to support services for teen mothers. C. Provide psychoeducation about the effects of alcohol on fetal development. D. Immediately report the client for suspected abuse of her unborn child.

A. D is outbecause you do not report child abuse on an unborn child. If you put the other options in order, A isthe best answer. It would be importantto assess her understanding of her behavior before intervening.

A therapist working at a community agency is meeting with a 16-year-old girl. The client reports that her parents have been arguing a lot lately and last week during a fight she tried to protect her mother, but instead was punched by her father. Agency protocol states that the social worker must consult with another therapist before making a report, but the social worker is unable to find an available colleague. What should the social worker do NEXT? A. Make a report to the appropriate authorities B. Hold off on reporting until a colleague can be consulted C. Call the client's mother to create an appropriate safety plan D. Assess further to determine the severity of abuse

A. Despite agency policy, it is still the responsibility of the therapist to make a report, since the client may be in immediate danger. B would put the client at risk. C would also be inappropriate, because it would rely on the parent to participate and does not take the client's immediate safety into consideration. D would be inappropriate, because the client has already expressed that abuse is occurring.

A social worker at a mental health clinic has started to notice changes in a colleague's behavior. Once a pleasant and thoughtful therapist, this colleague has become irritable and argumentative. At a recent case consultation, the colleague fell asleep in front of the medical director and appeared disheveled. What should the social worker do FIRST? A. Address the concerns privately with the colleague. B. Meet with a supervisor to address the concerns. C. Schedule a meeting to address the concerns with the colleague and a supervisor. D. Address the concerns with other co-workers before bringing it up with the colleague in question.

A. Ethically, you should start by addressing your colleague privately. If you continue to have concerns that your colleague is not addressing the issue, B would be the next option.

A client who is addicted to heroin is trying to quit on her own. She comes into your office and complains of nausea, sweating profusely, anxiety and severe muscle cramps. The next step should be to: A. Get the client medical assistance. B. Tell her to just wait and few days and the symptoms will subside. C. Refer her to the nearest Narcotics Anonymous group. D. Tell her to slowly cut back on her heroin use so she doesn't experience strong withdrawal symptoms.

A. The client needs to get immediate medical assistance and she should be monitored when she when she is going through heroin withdrawals.

In an initial session of family therapy, parents of two teenagers begin complaining about their children's attitudes, reporting that the two have no respect for their authority. The teenagers respond by reporting that "there isn't anything there to respect" and blame their bad attitudes on the parents. The social worker should FIRST: A. Take a neutral stance by acknowledging everyone's feelings. B. Have individual meetings with the parents and the teenagers. C. Use circular questioning. D. Empathize with the teenagers.

A. In family therapy, it is important for the therapist to develop a therapeutic alliance with all family members. B does not make sense as it splits the family up when the issue is a family dynamic. C may be useful at a later time to facilitate a conversation within the family but only after A has been done. D is inappropriate because that would not be taking a neutral stance and would be taking sides against the parents.

A 21-year-old male with a substance abuse disorder drops out of treatment after acknowledging that he has relapsed in a significant way. The client's mother contacts the social worker and begs the social worker to contact her son and try to convince him to come back to treatment. The social worker does not have a consent form to release information. What is the social worker's MOST appropriate response? A. Provide no information to the mother. B. State that the client has the right to refuse treatment since he is an adult. C. Encourage the mother to address her concerns with her son. D. State that the social worker is worried about her son as well.

A. It would be breaking confidentiality to do B, C, or D. A is the best answer.

A new client reports a feeling that people are "running her over" and notices she has trouble getting her husband to listen to her and eventually gives up. The client and social worker have started to work on treatment goals, but have made little progress. When bringing this up, the client reports that she is worried about being "too masculine" in front of her husband when she tries to assert herself, saying that her husband's culture really embraces femininity. What should the social worker do FIRST? A. Explore the client's beliefs around the cultural dissonance B. Encourage the client to refocus on her initial goals when she came to therapy in order to encourage self-determination C. Rework the goals with the client to make them more achievable D. Encourage the client to bring in her husband in for the next session

A. It's important to engage the client in the how she believes the cultural dissonance is affecting her ability to work towards her goals. B does not address the client's fears about her husband and C would ignore the client's initial reason for coming to therapy. D would not be the first option for the therapist to take, until the client's beliefs have been explored.

A therapist has been seeing a client for 6 months when the client reports that she lost her job due to funding. The client reports that her partner recently took a pay cut to start working at a non-profit whose mission she is passionate about. The client expresses worry around supporting their children now that they are only on one income. Which of the following should the social worker do FIRST? A. Prompt the client to explore how this is affecting their family B. Refer the client to an agency that assists people in finding jobs C. Assist the client in looking at her budget to see what areas can be cut D. Provide the client with referrals to financial aid resources

A. It's important to understand the hierarchy of needs in order to determine the next step in this case. The therapist needs to get a sense of how the loss of income is impacting the family and what needs are not being met before can move forward. Once that has been completed, the therapist can explore B, C, and D.

A 25-year-old client begins to cry in session while disclosing that her male gynecologist made several inappropriate comments about her body and genitals during a recent exam. She believes this was wrong, but asks the social worker not to disclose this information. What should the social worker do FIRST: A. Validate the client's feelings and explore the impact of this experience on the client. B. Report the doctor to the medical board. C. Point out how other women may be victimized by this doctor. D. Encourage the client to report the doctor.

A. Social workers need to respect clients' right to self-determination first; since this client is explicitly stating that she does not want the social worker to disclose the information, the social worker should respect the client's right to self-determination and confidentiality. B would be inappropriate due to the client's request. C and D both put pressure on the client to do something that she has said she does not want to do.

Which of the following statements is true of summative evaluations? A. They occur at the conclusion of a program and make judgments about the efficacy of the program B. They are done during the development of a program to ensure implementation of the program is done effectively C. They are completed throughout the implementation of a program to ensure improvements are made D. They include recommendations for program improvement

A. Summative evaluations (also known as outcome evaluations) determine the overall effectiveness of a program and occur at the end of a program. B and C are both incorrect because the timing and stated purposes are wrong. While D may occur as a result of the summative evaluation, this is not inherently included as a part of the summative evaluation.

A mother brings her 3-year-old daughter to therapy. The mother shares that she recently divorced her spouse and that she shares custody of her two kids with her ex-husband. The children stay with the mother during the week and are with their father on the weekends. Recently, her 3-year old daughter has started wetting her bed even though she was toilet trained before. She also cries when she needs to go to her father's house. What is the social worker MOST appropriate response? A. Tell mother that this is a normal response when dealing with parent's divorce. B. Find out if there are any child abuse issues going on. C. Ask mother about her relationship with ex-spouse. D. Take developmental history of the child.

A. The child's behavior is a normal reaction to the recent changes she has experienced. It would be good to start with A and normalize it for the mother. B, C, and D could all be done but if leading with those you could cause the mother to feel that there is something wrong with her child, when it fact this is a very common reaction to the stressors, which should be emphasized along with further assessment.

A social worker meets with a client who is diagnosed with Major Depressive Disorder and is also dealing with an addiction to prescription painkillers. The client is fearful of relapsing on pills during periods of stress and anxiety. To help this client cope with the urge to use drugs, the social worker should FIRST focus on: A. The client's coping patterns to deal with stress and other uncomfortable feelings. B. The client's social supports. C. Professional consequences of drug use. D. The client's motivation to change behaviors.

A. The client uses drugs to cope, so the best focus is on dealing with stress (A). The other answers won't give client tools to cope.

Social workers from a community counseling center are providing crisis intervention at a local middle school. A recent earthquake killed two students and a teacher while they were on school grounds. The students report feeling scared to come to school and anxious that another earthquake will occur at any moment. When meeting with students, the SW should FIRST: A. Validate the client's feelings and reassure them that their feelings are normal. B. Create a support group for the students. C. Develop a safety/evacuation plan with the students in the event of another earthquake. D. Provide the students with local mental health resources.

A. The clients have expressed their feelings. A would be the next step, because their reactions are normal. B and C could be done later and there is no indication for D at this point.

A family comes in with high expectations of their 10-year-old child. The child excels in school and the teacher states that he has never had an infraction although he appears to be unhappy and has a hard time socially. The parents are likely using what kind of parenting style? A. Authoritarian B. Authoritative C. Permissive D. Uninvolved

A. The clues in the stem are the child excels and hasn't had an infraction but seem unhappy and has a hard time socially. This is the description of a child raised in an Authoritarian home. Children raised with Authoritative parenting styles are the best adjusted. Both Permissive and Uninvolved children display behavioral problems and generally have lower school performance.

A new client comes for her initial session to a private practice. She is currently going through a divorce, which involves a custody battle of her 5-year-old son. She just moved to a new house, started a new job and is having relationship issues with her mother. The social worker should: A. Help the client prioritize needs. B. Meet with the husband to assist with the custody agreement. C. Assess the safety of the child. D. Obtain a release to talk with the husband.

A. The first thing to do would be to prioritize the client's needs. There is nothing to indicate that B, C, or D are needed at this point. Having the client prioritize needs supports self-determination and empowerment of the client.

The parents of a 7-year-old boy meet with a social worker due to concerns about their son's behavior. The parents report that the boy cannot sit still, does not follow directions, interrupts family members at the dinner table and does not have friends in the neighborhood. When asked about how the child behaves at school, the mother reports that she isn't sure. What should the social worker do NEXT? A. Have the parents give the social worker permission to speak with the boy's teacher. B. Schedule a session with the parents and the boy. C. Encourage the parents to have their son evaluated for a learning disability. D. Refer the parents to an educational parenting group.

A. The parents tell you that there is missing information. A is the best place to start; once that information is available the rest of the interventions can be determined.

What is the PRIMARY function of case management and the case manager? A. To advocate for clients and connect them to community resources B. To provide psychotherapy C. To investigate allegations of child abuse and neglect D. To help clients prioritize their presenting problems

A. The primary responsibility of case workers is to advocate for our clients and to help connect them to community resources (A). Only CPS case workers investigate allegations of child abuse and neglect, making C incorrect. While a case worker may help clients prioritize presenting problems (D), other social workers would do this as well and it is not their PRIMARY responsibility. In contrast to clinical work, case management doesn't involve engaging in psychotherapy. When doing case work, a social worker will still engage with the client, gather history/assess, and identify (and prioritize) the clients needs. Then, rather than engaging in psychotherapy/clinical interventions, the work itself involves identifying resources in the community (including psychotherapy services) and getting the client connected to those resources to meet their identified needs.

A 29 year old is in treatment for Schizophrenia. She complains to her social worker about experiencing involuntary facial tics and twitches in her arms and legs. What should the social worker do? A. Explain to her that she could be experiencing side effects from her antipsychotic medication and refer her to speak to her Dr. B. Assess her to see if she is using illegal drugs. C. Explain that this can be a symptom of Schizophrenia. D. Talk to her Doctor about increasing her medication since she is experiencing greater symptoms

A. The symptomsshe is experiencing are common side effects of antipsychotics. A is the bestbecause it refers her to talk with her Doctor about it as this would be out ofthe scope of practice of a social worker.

A social worker is conducting a study on the effectiveness of group therapy in decreasing depression in teenagers recently diagnosed with diabetes. Which of the follow refers to the extent to which the results of a study can be generalized to other situations and people? A. External Validity B. Internal Validity C. Reliability D. Statistical Significance

A. This is a factual recall question; B, C, and D would all be described differently.

Which of the following therapies is present-focused and short-term, that helps the client identify, challenge, and replace self-defeating thoughts and feelings that interfere with life goals with more productive, helpful behaviors that help with the attainment of goals and positive relationships: A. Rational Emotive Behavior Therapy B. Dialectical Behavior Therapy C. Humanistic Therapy D. Psychodynamic Therapy

A. This is arecall question. Rational Emotive Behavior Therapy is a short term therapy thathelps clients identify and replace self-defeating rigid thought patterns,beliefs, and unhealthy behaviors that interfere with their life goals withhealthier thoughts and behaviors that help them achieve their goals. Some toolsused in REBT include: cognitive reframing, visualizations, self-help tools, andhomework assignments. All other therapies listed would be describeddifferently.

A social worker receives a referral from parents regarding their 9 year-old daughter. A social history reveals symptoms consistent with a diagnosis of Major Depressive Disorder. What should the social worker do FIRST? A. Determine if the child has had a recent physical examination to rule out possible physical causes for her symptoms B. Under the diagnosis of Major Depressive Disorder, refer the child to a psychiatrist for medication. C. Have the parents fill out a questionnaire to confirm the diagnosis of Major Depressive Disorder. D. Begin cognitive behavioral therapy with the child to decrease symptoms.

A. While B, C and D may all be appropriate at some time in the near future, option A is the best choice as medical conditions should always be ruled out before giving a diagnosis.

A social worker at a local community mental health agency started a group for adults who grew up in the Foster care system. Each week fewer group members show up. In order to keep the group strong and ensure reasonable attendance, the social worker should: A. Talk to the group members to assess why attendance is dwindling and have them assist with the group structure and future topics. B. Cancel the group since it is not needed. C. Change the day and time of the group to hopefully make it more convenient. D. Have a different social worker take over the group.

A. You would start by asking group members why they haven't been attending the group. B, C, and D are possible responses once you ask the members about their experiences.

A social worker meets with a woman who is concerned about her marriage. During the intake, the mother reports that she divorced her daughter's "good for nothing" father five years ago and has since married a man who is always doting on her daughter. The woman works second shift at a local factory and leaves her daughter with her husband. The woman worries that her husband does not seem interested in her sexually anymore; she goes on to complain that her daughter has started putting on an alarming amount of weight and is not taking care of her personal hygiene. What is the MOST likely explanation? A. The stepfather is adjusting to his role as a primary caregiver. B. The couple is experiencing a typical decrease in intimacy as they adjust to married life. C. The couple's age difference has put the woman in a parental roll with her husband. D. The stepfather is sexually abusing the daughter.

All the risk signs for sexual abuse are there in the daughter's behavior. D is the best explanation.

A 23-year-old client with Down Syndrome receives monthly home visits from a case manager after a previous suicide attempt and hospitalization. The client lives in a group home and has a roommate. During the most recent visit, the case manager notes that the client appears depressed and can't provide information about recent psychiatric appointments, though the apartment appears relatively clean. The client's roommate enters the room as the social worker is leaving. The social worker should: A. Ask the roommate about the client's recent behavior and symptoms B. Exit the room without engaging the roommate in conversation C. Say hello and ask the roommate how things have been going D. Request that the roommate call the social worker if the client's symptoms worsen

B is the best answer, because it protects the client's confidentiality. A and D both violate confidentiality; C is a boundary violation, as it engages the client's roommate in social conversation.

A social work supervisor meets with a supervisee who shares that they are sexually attracted to a client they have been seeing for several months. What should the social work supervisor do FIRST? A. Work with the supervisee to get the client transferred to a different social worker B. Process the social worker's feelings of attraction toward the client C. Help the social worker strategize how to bring up their feelings to the client D. Discuss the need to report the supervisee to their state board for a boundary violation

B is the FIRST thing the supervisor should do with their supervisee. (A) may or may not be necessary depending on whether the supervisee is able to manage their feelings toward the client. If this were a brand new client, A could occur, but since they have an established relationship you would only want to refer out if the supervisee were unable to manage their feelings. (C) is not appropriate and should not occur. There is no need to report the supervisee (D) at this time, as no inappropriate actions have taken place.

A social worker meets with a couple who have been married for five years. The wife reports that she recently discovered that her husband has been having an affair with another woman. She says that she is angry and is considering filing for divorce. The husband states that he is sorry and wants to do what he can to save their marriage. After the session, the social worker feels some anger towards the husband since she went through a similar experience in her own marriage. What should SW do NEXT: A. Explore the events leading up to the affair. B. Schedule a meeting with a supervisor to address possible countertransference issues. C. Encourage the couple to each receive individual counseling. D. Disclose the personal experience to show a sense of shared experience.

B is the best answer of the options. The social worker is having a countertransference reaction to the couple that should be addressed. A and C may be helpful at a later point. D should only be done with careful considerations to the clinical benefit.

A gay couple who say they are both in long-term recovery for cocaine addictions request couples' therapy. The couple reports frequent arguments and an increased sense of distance in their relationship. The social worker should FIRST: A. Teach the couple communication skills. B. Assess for a drug relapse in either partner. C. Assign the couple intimacy exercises as homework. D. Teach the couple anger management skills.

B is the best answer, because a decrease in intimacy and an increase in arguments may be signs that one partner has relapsed, putting stress on the relationship. This needs to be assessed first. A, C, and D can be done later.

A 12-year-old African-American boy has been meeting with a social worker for 2 months for feelings of depression. He recently discovered that one of his ancestors was a slave who led a rebellion during the Civil War. He previously felt ashamed of his background, but in sessions, starts speaking about his ancestor as being a "brave leader." In terms of racial identity development, the client's experience could be best described as: A. Conformity B. Dissonance C. Resistance D. Introversion

B is the best answer, because according to a minority identity development model, the stage of dissonance includes a period in which the individual moves from feeling shame about their identity to feelings of pride. In the stage associated with A, the individual prefers the dominant culture and devalues their own background; in the stage associated with C, the individual rejects the dominant culture and has a sense of appreciation for their own culture; in the stage associated with D, the individual moves toward feeling that the dominant culture is not all bad and that there are individual differences.

A client with a history of interpersonal problems with co-workers has recently been fired for getting into repeated arguments with colleagues and supervisors. The client arrives at the next session late and becomes very angry when the social worker inquires about the client being late. The client yells, "Why are you monitoring my every move? Can't you just trust me and leave me alone?" The social worker should interpret the client's behavior as: A. Resulting from a feeling of judgment by the social worker. B. Displaying problems that occur in workplace relationships. C. Indicating a need for medication to stabilize the behavior. D. Signaling a desire to terminate treatment.

B is the best answer, because it is MOST likely that the client is exhibiting behavior that is typical in another area of the client's life; since the question references problems in the workplace, this is probable. A, C, and D are not supported.

A social worker in a community mental health agency begins working with parents and their 12-year-old son. The family has been referred because the son's grades are deteriorating and he's been getting into fights at school. During the first few sessions, the social worker notices that the parents repeatedly express anger toward their son for lying, but when the son tells the truth, they yell at him for his "poor choices." The social worker should NEXT: A. Separate the child and the parents to help them each express frustration in a safe space B. Point out this double bind to the parents C. Ask the son to respond to the parents' anger D. Point out how unproductive being dishonest it

B is the best answer, because it names the dynamic that is occurring in the room and may help the parents understand why the boy is not likely to be honest with them. A is not indicated in this situation and will not help address the dynamic that is occurring. C may be helpful, but not before the social worker helps raise the parents awareness of the dynamic. D is not supportive and will also not help address the dynamic at hand.

A social worker in a community service organization receives a referral for a 17-year-old girl who was recently kicked out of school due to poor attendance. When the girl and her mother arrive, the mother tells the social worker that the girl recently immigrated from Venezuela and does not speak English. The mother immigrated several years ago and can communicate clearly. The social worker should NEXT: A. Use her limited Spanish to assess the client B. Tell the mother that the appointment will be rescheduled with an appropriate interpreter present C. Ask a Spanish speaking staff member to interpret the session D. Allow the mother to interpret during this session and schedule an interpreter for follow up appointments

B is the best answer, because it provides the client and social worker with the appropriate level of support. A is inappropriate, since the social worker is not fluent and may not be able to communicate or understand the client effectively. C and D are both inappropriate, because the client should have an interpreter who has been specifically trained to serve this purpose and will provide unbiased, unfiltered information.

A social worker is meeting with the parents of a 7-year-old boy who was recently diagnosed with ADHD. The parents met with a psychiatrist who recommended medication and behavioral management therapy. The parents are resistant to medication. What should the therapist do first? A. Refer them back to the psychiatrist. B. Explore their concerns about medication. C. Begin behavioral modification training. D. Encourage them to try out medication before they judge it.

B is the best answer, because it's important for the social worker to explore the parents' concerns about medication and what is behind that concern before further steps are taken.

A patient admitted into ER has his partner listed as the emergency contact. The social worker learns that the nursing staff contacted his family instead of the partner. The social worker should NEXT: A. Call the partner. B. Tell the patient that his family was notified. C. Wait until the family arrives and find out what they think is best. D. Discuss the situation with the doctor.

B is the best answer, because the client should know and be able to emotionally prepare for his family's response or arrival at the hospital. A might be a good step, but not until after the client is informed of the mistake. C would violate the client's confidentiality and take away his self-determination. D does not make sense, since the doctor is not mentioned in the stem and has not been a part of this particular situation.

A 50-year-old woman in the commercial real-estate business is referred to her EAP due to interpersonal conflicts with female co-workers. During the assessment, the social worker learns that the woman frequently minimizes her colleagues concerns regarding sexism and sexual harassment in the workplace and during work functions. The woman says, "They need to get over it. This is just part of the business. They're being too sensitive." The social worker should view this response as: A. Expected, given the employment setting and cultural standards. B. An internalization of the sexism the client has experienced during her career. C. Evidence of a personality disorder. D. An indication that long-term therapy is needed.

B is the best answer, because the client's behavior indicates she has internalized the sexism she has experienced over time. A is not a good answer, because, while it may be true, it does not identify the process through which this has occurred as well as B does. C is not indicated by the client's statements or behavior. D may be true, but is not clear from the information provided in the question stem.

An adult with Down Syndrome and diabetes is in an emergency room for treatment due to noncompliance with insulin shots. Before releasing her, the physician asks the social worker to interview the client and her parents, with whom she lives, about the situation. The social worker learns that the noncompliance with insulin has been an issue at home and at the client's place of employment. What should the social worker do FIRST? A. Provide the parents with information about how to apply for conservatorship. B. Explore the client's understanding of her illness and the significance of medical treatment. C. Discuss the effects and benefits of medication with the whole family. D. Reframe the client's noncompliance as an example of her seeking independence

B is the best answer, because the social workerfirst needs to understand what is behind the client's noncompliance beforemoving forward. A is too extreme andlikely unnecessary. C is a goodintervention but not before B; D is not supported by the information in thestem

A social worker has been meeting with a 14-year-old boy for the past month after he came out as gay to his classmates. During a recent session, the client and therapist process his desire to start dating and identify steps he can take that feel comfortable for him. At the end of the session, the client gets up to leave and blurts out, "I went to a party last week and had sex with someone I've never met." This statement is BEST described as: A. Developmentally typical behavior B. Doorknob communication C. Manipulation to extend the session D. An expression of distress around his sexual identity

B is the best answer. When a client abruptly tells the social worker something significant at the very end of a session, it is referred to as a "doorknob communication." A and C could be true but are not as accurate a description as B. D is not indicated by this statement.

A young mother brings her 2-year-old son to the emergency room for treatment of burns to the hands and face. The mother states that the injury occurred when the child pulled a cup of hot coffee off of a table when the she was not looking. The medical report shows third degree burns to the hands and face, as well as several bruises on the child's buttocks and back that are in various stages of healing. After hospitalizing the child, the medical staff makes a report of suspected child abuse. The child welfare social worker should FIRST: A. Remove the child from his home and arrange for alternative care. B. Follow through with the report of suspected abuse and conduct an investigation. C. Refer the mother to a parenting class. D. Encourage continued hospitalization for the child.

B is the first place to start. A and C can be done later. D is outside social work scope of practice.

A couple in their 60's meet with a social worker and state that they are upset that they have had no sexual relationship during the past year. The social worker should FIRST: A. Address other stressors in their marriage. B. Address underlying reasons for the lack of sexual activity. C. Explain that most couples stop being sexually active as they get older. D. Encourage the couple to see a sex therapist.

B starts where they are and addresses the presenting problem. C is inaccurate. A and D could be done later

A woman has been going to therapy for the past two months. Whenever the therapist brings up emotional material the woman dismisses it and says she isn't interested in talking about those subjects. She talks a lot about her day to day life and says she thinks she is ready to terminate. What should the therapist do next: A. The client seems like she is doing fine and feels she is ready to terminate so the therapist should start the termination process. B. The therapist should address the client's resistance in discussing the emotional material. C. The therapist should give the client two more sessions to see if she still feels ready for termination. D. The therapist should inform the client that she needs to talk about this material or else the therapy sessions will be over.

B. A client expressing disinterest in discussing emotional material is likely a sign of resistance.

A 45-year-old man meets with a social worker for an initial appointment. He reports that his wife is an alcoholic and becomes verbally abusive when she is intoxicated. He indicates that his wife refuses to go to treatment, insisting that they can resolve this problem on their own. The therapist should FIRST: A. Provide the man with psychoeducation around substance abuse and the stages of change. B. Explore the man's issues and work collaboratively to develop treatment goals. C. Explore the man's commitment to the relationship. D. Help the man identify the reasons behind his wife's behavior.

B. A, C, and D could be options later in treatment after a treatment plan focusing on the man has been established.

A 23-year-old woman seeks treatment from a social worker through her university following a dramatic breakup with her girlfriend. During the assessment, the social worker notes that the client seems to have abandonment issues that stem from a long history of fragmented relationships. If the social worker operates from a client-centered therapy model, all of the following interventions apply EXCEPT: A. Empathizing with the client's experience B. Exploring the client's cultural background and expectations in relationships C. Matching the tone and pacing of the client's verbal communication D. Maintaining a positive stance toward the client

B. A, C, and Dare all interventions that are associated with client-centered therapy. B isnot specifically associated.

A woman brings her 30-year-old daughter to meet with her social worker. The daughter is diagnosed with depression and lives with her mother. The mother reports that the other night, the daughter made a suicidal threat and then locked herself in the bathroom for 2 hours; the mother is now afraid that the daughter is ready to make a suicide attempt. The daughter has two previous suicide attempts and has stopped taking her antidepressants. In the interview, the social worker finds that the client is withdrawn, but calm, and unwilling to contract for safety. What should the social worker do FIRST? A. Help the mother contact the daughter's psychiatrist. B. Begin to take steps towards an emergency psychiatric commitment. C. Attempt to convince the daughter to take her medication. D. Explore the daughter's recent increase in suicidal thoughts.

B. Because the client is unwilling to contract for safety, the therapist must consider B. Her safety is the most important issue and her unwillingness to sign contract is an indication of high risk for suicide

A social worker is attending a neighborhood block party. While speaking with her neighbor, the neighbor discloses that she was so frustrated with her child the other day that she hit her with a belt. While the neighbor is not a client of the social worker, the social worker should first: A. Make a suspected child abuse report B. Report the incident according to state law. C. Refer the neighbor to a different social worker so there is no dual relationship. D. Contact the school to discuss the child.

B. In some states, social workers are mandated reporters 24/7, while in others, they are only mandated reporters while in their professional capacity. A would not be correct, since the law may not require this. B is the best answer, because it allows the social worker to do whatever is legally required. The social worker could do C if the neighbor asked for assistance. D would violate the neighbors trust.

A 30-year-old client meets with a social worker and states, "I feel like I'm driving myself insane." The client reports a fear of germs to the point where she won't allow others to visit her home because she is afraid they will bring germs into the home. She says she rarely leaves her home and spends a lot of the day cleaning. The client also states that she scrubs her body to the point where she is bleeding. The social worker should FIRST: A. Suggest client keep log of what triggers her behaviors. B. Provide the client with a referral for a medical evaluation. C. Help the client determine hand washing alternatives. D. Identify the cause of the phobia.

B. Medical evaluation would be the place to start due to the severity of the symptoms. A and C are interventions that could be done later. D is not relevant to the treatment of phobias.

A mother brings her 10-year-old daughter to therapy after an unfounded abuse investigation on the girl's father. Since the investigation, the child has been afraid to sleep in her own room, is demanding of her parents, and has nightmares that her dad is being taken. What should be the immediate, short term objective of therapy with this client? A. Encourage the client to forget the incident because it was unfounded. B. Assist the client to explore her emotions and fears about the incident. C. Instruct the parents to set firm limits on child's bedtime behavior. D. Refer the client to a psychiatrist for a medication evaluation.

B. Since the stem is addressing which short term objectives should be developed, exploring the client's emotions and fears is the best place to start. A would be inappropriate and invalidating. C and D may be helpful to explore later.

A social worker is providing therapy for a woman just out of rehab and her 12-year-old daughter. At a session with the daughter, the daughter reveals that she is having a hard time sleeping because her mother is abusing drugs again and states that her mother sleeps at her boyfriend's house and leaves her and her 4-year-old brother home alone. The social worker should FIRST: A. Clarify what types of drugs the mom is using. B. Report the child neglect situation to the appropriate agency. C. Schedule a joint session with the mother and daughter. D. Find out where the boyfriend lives.

B. Social workers are mandated to report suspected child abuse and neglect. A is inappropriate, since it goes beyond reporting to investigation; C is inappropriate, since the 4-year-old boy's safety may be in jeopardy; D does not make sense.

The parents of a 6 yo boy who was diagnosed with Autism Spectrum Disorder meet with a social worker. During the session, the parents complain that the school does not provide a classroom that caters to their child's needs. The social worker should: A. Advocate for the child by speaking with the school administration. B. Support the parents in scheduling a meeting with the school administration. C. Schedule a meeting with the child's teacher to discuss a behavioral plan. D. Provide the parents with behavioral suggestions that can be implemented at home and shared with the teacher.

B. Social workers should empower clients to advocate for themselves. A and C do the work for the clients. D comes after B.

When a potential client inquires about services, the MOST important reason for addressing the various functions and services of the agency is to: A. Address the limitations of agency services. B. Explore if the client's needs can be addressed by the agency's services. C. Explore whether they fit with the client's expectations. D. Determine whether the client is eligible for services at the agency.

B. The client's needs come first so B is the best answer. The other answers would be considered but are not as important in responding to a potential client

A hospital social worker is tasked with discharge planning for clients who have been involuntarily committed for non-lethal suicide attempts. The social worker links clients to mental health agencies within the community to ensure their individualized after-care needs are met. What should the social worker do NEXT? A. Nothing; once the clients have been discharged no follow up services are needed B. Utilize a care coordination model to determine whether clients have been effectively connected to community services C. Gather a team of professionals to decrease the rate of suicide attempts in the community D. Create a list of community referrals to provide clients upon discharge

B. The hospital social worker is charged with discharge planning and is connecting clients to agencies within the community. Once this has occurred, clients need to be followed up with to determine whether they have sufficiently connected to the services identified during discharge planning, which (B) does. A is incorrect; after they are discharged, follow up services are often needed. (C) does not connect to discharge planning and D is insufficient, as it does not assist the client in navigating resources following their discharge.

A client meets with a social worker and states that she has a medically frail child who is 3-years-old. The client states that she feels hopeless that things will change, and is angry at how it's affecting her family. The client feels that the needs of this child are impacting the client's other children who are oppositional and having problems in school. What should the SW do FIRST? A. Refer the client to a support group for parents of children with medical problems. B. Work with the family as a unit. C. Coordinate with school social worker to address problems with other children. D. Work with the client around ways to reduce her stress.

B. The problem is affecting the whole family, so that is the best method of treatment. A, C, and D could all be done as part of the treatment.

A social worker meets with a mother who has two young children with severe asthma. One of her concerns is the mold in their apartment unit that has become increasingly worse. In order to advocate for the client, the social worker should: A. Obtain a release to speak to their pediatrician. B. Support the mother in speaking with her landlord regarding her concerns. C. Determine if other neighbors share their concern. D. Provide psychoeducation about mold removal.

B. The question is asking about advocacy, which means trying to make change. The first step would be B. A and D are about dealing with the incident, not about change. C could be done later if the landlord was unresponsive.

A social worker has been working with a 24-year-old client who has recently lost his job and is depressed; the client has had to move back in with his parents as a result of lost income. One day, the client's mother calls the social worker and expresses concern about her son's inability to find a new job. The mother presses the social worker to learn whether or not her son has discussed plans to contact a staffing agency for an interview. The social worker should FIRST: A. Validate the mother's concern about her son. B. Explain that information cannot be disclosed without a client's consent. C. Discuss the mother's concerns with the client at the next session. D. Set up a family session with the mother and son to discuss their issues.

B. The social worker MUST ensure the client's confidentiality; A and D would both violate the client's confidentiality; C may be appropriate at the next session, but would not come before the therapist declined to disclose information to the mother.

A social worker in private practice meets with a client who is working through issues related to his father's recent death. During one session, the client talks of feeling overwhelmed by his feelings of grief and unable to deal with his own son's crying and sadness about the loss. The client reveals becoming so frustrated with his son over the weekend that he grabbed the child forcefully and left bruises on the child's arm. The social worker should NEXT: A. Assess the child. B. Let the client know that the child protection agency must be notified about the incident. C. Educate the client on the stages of grief and provide him with relaxation skills. D. Refer the client to a grief support group.

B. This is a mandated report. B must be done first. A, C, and D could be done later.

When a clinical supervisor interacts with their supervisee in a way that models for the supervisee how they should interact with their clients, this can be referred to as: A. Countertransference B. Parallel Process C. Transference D. Mentoring

B. This is afactual recall question. Parallel process occurs when a supervisor interactswith a supervisee in a way that models through the supervisory relationship howthe supervisee should interact with their clients.

A social worker meets with a lesbian couple seeking help with the decision of getting married and starting a family. After two sessions, the social worker asks questions regarding their sexual relationship. The clients appear uncomfortable and ask why the social worker wants to know. The social worker should first explain that: A. It is common for couples to experience sexual problems. B. Exploring their sexual history is an important aspect of addressing underlying issues. C. Beliefs surrounding sex are an important part of the relationship to address. D. It is important to address any sexual problems now before they escalate.

C is the best answer and is related to why they are coming for therapy. A, B, and D suggest there is a problem.

A 16-year-old is brought to treatment for an eating disorder. How would a social worker using Feminist Theory Therapy view this issue? A. Unresolved issues of early childhood can manifest as eating disorders. B. Her automatic thoughts cause unhealthy body-image. C. Cultural expectations of women negatively affect their sense of self. D. Denial of food is a defense mechanism to manage anxiety.

C is the best answer because it directly relates to Feminist Theory and how mental health is affected by a patriarchal culture. A would be more of a psychodynamic view. B would be a Cognitive Behavioral view. D is not as specific but is more psychodynamic in nature.

A 41-year-old woman has been in therapy for two years for anxiety attacks and depression. She is no longer taking medication and is currently symptom free. During recent therapy sessions she frequently laughs and enjoys herself. Which of the following actions should the social worker take? A. Interpret the client's behavior as a flight into good health. B. Confront the client regarding avoidance of underlying issues. C. Develop a termination plan with the client to maintain progress made. D. Identify precipitating events leading to the new symptom development.

C is the best answer because the client's presenting problems have been resolved and the client is doing well without the aid of medication, both of which indicate a readiness for termination. A does not make sense because the client has been in treatment for 2 years; a flight into good health is typically seen at the initiation of treatment. B and D are not supported by the information in the stem.

The drug Antabuse uses which behavioral modification principal to keep people from drinking alcohol? A. Positive Reinforcement B. Negative Reinforcement C. Punishment D. Classical Conditioning

C is the best answer, because Antabuse makes the person sick if they ingest alcohol, effectively providing a negative consequence for the behavior. A provides a positive consequence in response to the desired behavior. B removes a negative consequence in response to the undesired behavior. D utilizes a naturally occurring stimulus paired with a desired response; then a neutral stimulus is paired with the naturally occurring stimulus. Finally, the neutral stimulus is used alone to prompt the desired response.

A social worker in a community clinic meets with a 27-year-old woman who was referred after experiencing domestic violence. The woman shares that she is pregnant and plans to return home after staying with her sister while her boyfriend "cools down." The social worker should FIRST: A. Convey acceptance of the client's decision to return home B. Report the client to CPS for child endangerment C. Provide psychoeducation around the potential escalation of domestic violence during pregnancy D. Express concern about the client's plan to return home

C is the best answer, because it helps the client understand the potential consequences of returning home while pregnant. While the social worker might do A at some point, it would not be done before discussing the potential for the violence to escalate. B is not appropriate since we do not report on unborn children. D is coercive and may negatively impact the client-social worker relationship.

A social worker and client have been working together for 2 months. The client, a 27-year-old woman, initially presented due to concerns about her alcohol use and its effect on her relationships and ability to perform her job duties. After the initial session, the client began minimizing her alcohol use and typically comes to session with another topic she wants to discuss. The social worker has attempted to confront the client about her avoidance of the topic, but the client is adamant that her drinking is not a problem at this time. The social worker should: A. Continue to confront the client about her alcohol use B. Consult with a colleague about the client's resistance to change C. Convey acceptance and monitor the client's readiness for change D. Discuss termination due to lack of client engagement

C is the best answer, because it meets the client where she is and also monitors when she might be ready to address her substance use. A is not the best answer, because the social worker has already attempted to confront the client and it was not effective. Continuing to confront the client may damage the therapeutic relationship. B might be appropriate but is not as good as C in terms of meeting the client where she is. D is not indicated: the client keeps coming to treatment and may be benefitting from it in many ways.

A social worker in a community mental health agency meets with a 14-year-old boy and his parents after 3 months of treatment. The family first presented for treatment because the boy was experiencing feelings of anxiety and had several panic attacks. The goal of the session is to review the client's progress and reevaluate the treatment plan. All of the following are examples of objective data EXCEPT: A. The number of panic attacks the client reports in the past 2 weeks B. The client's score on the Beck Anxiety Inventory C. The client's ability to express feelings to the social worker D. The parents' report on how often the child avoids anxiety-provoking situations

C is the best answer, because it represents subjective data (information from the client's point of view gathered through an interview). A, B, and D are all examples of objective data: observable and measurable data.

A 38-year-old woman with Borderline Personality Disorder continually engages in scapegoating during a weekly treatment group. After consultation, the social worker facilitating the group decides to dismiss the client from the group due to the destructive nature of her in-group behaviors. What should the social worker to FIRST in order to facilitate termination? A. Inform the client during a group session that she will be leaving the group in 2 weeks. B. Ask the group whether the client should be dismissed from the group. C. Schedule an individual appointment with the client to discuss the decision. D. Refer the client for individual treatment before termination is discussed

C is the best answer, because this is a matter that should be handled privately and confidentially. A and B are inappropriate and would clearly shame the client in front of other group members. D could be done as part of the individual appointment, but would not be done FIRST.

A client is referred for therapy by his wife. He shares during the assessment that his wife has threatened to divorce him if he doesn't address his drinking problem. The client states that he doesn't think he has a problem with alcohol but doesn't want to lose his wife. The social worker should FIRST: A. Refer the client to detox. B. Obtain a release to speak to the wife. C. Explore the specific events that precipitated his entry into therapy. D. Confront the client's denial of his drinking problem.

C is the best answer, because without understanding the precipitating events, the social worker will be unable to accurately assess the problem. A and D both assume that he does, in fact, have a drinking problem, which may not be the case. B may be an appropriate action, but not until the social worker understands the client's perspective and what led to him being there.

A mother brings her six-year-old in to see a social worker after a teacher expressed concern regarding an incident at school. The teacher stated that the child has been taking dolls and mashing their faces and bodies together, saying "they're doing it so they can make a baby." This child is MOST likely: A. Being abused. B. Expressing developmentally-appropriate sexual impulses. C. Illustrating sexual behaviors that have been observed. D. Struggling to accomplish the psychosocial tasks of the identity vs. role confusion stage.

C is the best answer. The best explanation is that the child has been exposed to sexual behavior and had it explained to him. There is no indication of abuse in this stem. This behavior is not "age-appropriate," but it isn't a sign of abuse. Identity vs. role confusion occurs between the ages of 12-18, and is therefore inaccurate.

The mother of an eight-year-old boy calls the school social worker and says that her son has been refusing to come to school because he is teased about being overweight. The mother demands that the school punish the students who have been teasing her son. What should the social worker do FIRST? A. Encourage the mother not to overreact to the child's report. B. Get a list of the names of the bullies from the mom. C. Meet with the child to assess and develop problem-solving strategies. D. Refer the mom to a parenting support group.

C is the best place to start. You want to connect with your client (the child) and find out what is going on from him directly before you proceed with any other action. A dismisses the mother and the child's feelings; B disempowers the client. D might be helpful at some point but not before C.

A 35-year-old accountant seeks therapy because she has a hard time in social settings. She presents with a depressed mood and shares that sometimes she stays home and drinks rather than going out with her friends. The social worker should FIRST: A. Refer the client to substance abuse treatment for an assessment. B. Provide psychoeducation about how her substance abuse could be affecting her mood. C. Determine the frequency and amount of client's substance use. D. Ask if client experiences any symptoms of tolerance or withdrawal.

C should be done first because the client is presenting with many signs of substance abuse. Therefore the social worker needs to further explore the client's drinking habits. A, B, and D can only be done once C has been explored.

A client expresses concern about his 80-year-old mother's excessive drinking to a social worker. The client wants to set up an intervention to confront his mother about the problem. The social worker's BEST response is that the intervention: A. Will be ineffective due to his mother's age. B. Needs to include extended family members to be most effective. C. Needs to include a few individuals who are close to the client's mother D. Will only be successful if the social worker is present to facilitate the intervention.

C. A would not be a good response, so that can be ruled out. It is not necessary to include the social worker or extended family, so B and D are out. C is the best answer by default.

A social worker receives a letter from a personal injury lawyer requesting information about one of the social worker's client's' diagnosis and treatment. The letter indicates that the client is the defendant in an upcoming court case. What should the social worker do FIRST? A. Contact the lawyer and explain that a subpoena will be needed to release information. B. Contact the lawyer and release selected information from the client's record. C. Do not respond to the letter and contact the client to inform them of the request. D. Send a letter to the lawyer refusing the request.

C. A, B, and D all acknowledge that the client is the social worker's client, which is a violation of confidentiality. C is the only option.

A baby born with spina bifida has no use of her lower limbs and appears to have visual and hearing deficits as well. The hospital pediatrician refers the 16 year-old mother and her parents to the social worker. The social worker should FIRST explore the: A. Family's financial stability. B. Daughter's lack of prenatal care during early pregnancy. C. Family's attitude to caring for a child with special needs. D. Family's access to community resources.

C. B makes the assumption that the mother has done something wrong; A and D may be appropriate at some point, if it becomes clear that there is financial difficulty and a need for community resources but would not be done FIRST. C is the only answer that provides the family with an opportunity to explore their feelings around a difficult situation.

A client with a history of severe depression reports hypersomnolence and decreased appetite. The client reports having discontinued his prescribed medication because he didn't think he needed it anymore. The social worker should FIRST: A. Question the client as to the other reasons for discontinuing the medication B. Instruct the client to resume the medication C. Encourage to the client to speak to his psychiatrist D. Discuss alternative treatment options

C. Because the increase in symptoms is likely to be directly related to the discontinuation of medication, the client should be encouraged to go back to his psychiatrist. It would be outside of the social worker's scope of practice to instruct the client to resume medication. While questioning the client about the reasons for discontinuing medication and discussing alternative treatment options may be appropriate interventions at some point, the social worker should FIRST encourage the client to speak to his doctor regarding his increase in neurovegetative symptoms.

A social worker at a non-profit organization is preparing a proposal for a community education program around voter registration and rights. Which of the following activities is the social worker likely to engage in FIRST: A. Determine potential positive/negative outcomes of the program B. Identify educators to implement the intervention C. Conduct a needs assessment in the community D. Set goals and objectives

C. Before the social worker can identify A, B, or D, he/she should assess what the needs of the community are; everything else will be developed based on the specific needs of the group the program is set to address.

A social worker in private practice receives a referral for a five-year-old child who is having behavior problems at school and home. During the initial interview with the parents, they disclose that the child has a different biological father than his 3 older siblings and that everyone in the family knows about this except the five-year-old. The parents request that the social worker disclose this to the child. The social worker should FIRST: A. Meet with the child individually to assess his behavior problems. B. Tell the child the truth during an individual session. C. Meets with the parents to understand why they want to tell their child the truth now. D. Encourage the parents to disclose the truth.

C. C is the best because the social worker should collect more information about the presenting issue. A, B, or D could all be done later. Exploration is often a good place to start.

A single father seeks the assistance of a social worker in a family service agency. He reports increasing stress and difficulty sleeping, which have increased due to financial difficulties related to his mortgage. In the initial session, the client asks the social worker who will have access to what he talks about in session. The social worker's MOST appropriate response to him is: A. Request that he sign a release of information. B. Reassure him that confidentiality will be maintained. C. Explain the agency policies about confidentiality. D. Elicit information about why this issue has been identified.

C. Clients should be informed before treatment begins about confidentiality and its limits. A does not make sense since there is no indication that the client wants to release information to a 3rd party; B is inappropriate since there are some cases in which confidentiality cannot be maintained; D is inappropriate since it is the client's right to understand who will have access to his information and why.

An agency has chosen to use a single subject research protocol to assess their postpartum depression support groups. Which of the following BEST describes why an agency would choose this research model? A. It has strong external validity B. More than one variable can be measured C. Avoid ethical issues related to creating a control group D. Baseline measures are not required

C. Having a control group means that some individuals are not receiving the treatment. Withholding treatment from some individuals has ethical implications, making (C) the best answer.

A social worker is an instructor at a local social work program. One of her students is also an intern at the social worker's agency. The social worker knows that the student recently had a client die in a random accident. The student approaches the instructor and requests an extension on a paper that is due. The instructor should: A. Deny the request. B. Grant the request. C. Treat her like any other student. D. Suggest she drop the class until she can focus.

C. In cases where a dual relationship is occurring, the social worker should make sure to treat the student the same as any other student in the class (C). Therefore, a paper extension should only be granted if the teacher's policy allows for that.

A man has been hospitalized for a month following a car accident that has resulted in him needing a wheelchair while he relearns to walk. The patient's elderly parents would like the client to move in with them so they can help him while he regains his mobility to which the client agrees. In developing discharge plans, the social worker should FIRST evaluate which of the following factors? A. Financial stability. B. Access to physical therapy and community resources. C. The parent's ability to provide adequate care to their son. D. The accessibility of the parents' home for a wheelchair.

C. It is most important that the social worker evaluate how well the parents would be able to care for their son. Once that has been determined, the social worker can then assess other factors.

A social worker is about to meet with a woman who is experiencing difficulties in relationships. The woman is a single mother with a 5-year-old and has recently started to date a man that she met through AA. When meeting with the woman, what should the social worker do FIRST? A. Address the impact that being a single mother has on her relationships. B. Explore the woman's substance abuse history. C. Explore the woman's reason for entering therapy now. D. Schedule a session with her partner.

C. It is not clear why the client is coming for therapy, so that would be the first thing to determine. Once that is known, the focus of treatment can be decided.

A family seeks therapy for their 16-year-old son. He was diagnosed with Bipolar disorder six-months ago. The parents report that when he takes his medications he is able to function and attend school. Unfortunately, he frequently forgets to take his medication and has a manic episode. They are concerned that he won't be able to graduate and function as an adult. The social worker should initially focus on? A. His school performance. B. Creating healthy boundaries between the parents and son. C. Increasing compliance with medication. D. Identifying an alternate high school.

C. It's important that the social worker address the client's medication compliance FIRST before addressing other issues. Once the client is more stable, it may help him tackle some of the other issues that his parents are concerned about.

A married couple meet with a social worker in private practice for an initial session. Afterwards, the wife calls the social worker and states, "I have something I want to talk to you about, but my husband can't find out." The social worker's MOST appropriate response would be to: A. Reassure the wife that what she says will be kept confidential. B. Recommend that both husband and wife meet individually before the next couple's session. C. Advise the wife of the therapist's "no secrets" policy and suggest disclosure at a future couple's session. D. Refer the husband and wife to separate individual therapists.

C. Redirecting the wife to reserve the disclosure for the marital session is the best answer, because it will preserve the therapist's ability to treat the couple effectively. A would potentially jeopardize the couple's treatment by creating a situation in which the social worker and wife are keeping secrets from the husband. B does not make sense since couple's treatment has already been initiated. Recommending that the couple see individual therapists may be missing the point, as the couple has presented for marital therapy, not individual therapy, and keeping secrets may be part of the couple's problem.

An interdisciplinary team is struggling with group cohesion. Which of the following factors should be explored FIRST? A. Whether roles are correctly assigned B. How group leadership is impacting the team C. Whether trust has been established D. The length of time the group has been working together

C. Similar to building rapport with our clients, when working within a team we must first ensure trust has been established (C). A, B,and D could all be impacting group cohesion and rapport, but would not occur before first exploring whether trust has been established amongst the team.

A 72-year-old was brought into the ER by his daughter. The daughter reports that recently her father has started to leave the oven on, refuses to leave the house, and frequently has loud outbursts. She reports that today he became increasingly agitated and threw a knife at the wall while she was making dinner for him. What should the social worker do NEXT? A. Gain an understanding of the client's personality before the change occurred B. Explore the daughter's feelings around her father's decline C. Refer the client for a medical and psychiatric evaluation D. Explore assisted living home options with the client and his daughter

C. Since the client has recently had a change in mental status, it's necessary to have the client assessed for a medical and psychiatric evaluation. A is not important in having the client's immediate safety needs met first. B may be important for the social worker to do once her father has been assessed; however, an assessment needs to occur first. D may also be an option at a later point to ensure the client's safety.

After a session in which a client shared frustration about the social worker's approach, the social worker fears having made a mistake by dismissing the client's feedback. What should the social worker do during the next session? A. Admit that it could have been handled better. B. Focus on the positive aspects of therapeutic relationship. C. Bring up the incident and explore client's feelings about it. D. Wait and see if the client seems bothered by it.

C. Social workers provide a model for clients around how to discuss uncomfortable feelings and situations. A is inappropriate because it focuses more on the therapist's feelings; B is inappropriate because it ignores the situation and sets a poor precedent for the relationship; D puts the burden on the client to bring up the situation.

A social worker in a community agency receives a referral for her friend's preschool-age son. The social worker should: A. Involve the friend in a comprehensive assessment and treatment plan B. Conduct an initial assessment and make treatment recommendations C. Refer the son to a different therapist D. Treat the son after getting consultation regarding confidentiality and boundaries

C. Social workers should avoid dual relationships with clients. The son would be at risk of not receiving the best treatment, as the social worker may have preconceived notions about the son or may treat him differently due to the social worker's relationship with the mother. Avoiding dual relationships ensures the client's safety and ability to receive appropriate treatment.

A family seeks therapy for help with their teenage children. The parents recently got married and moved in together. The husband's children are ages 7 and 13. The wife's children are 9 and 14. The parents report that the teenagers are not listening and acting out. The social worker's initial intervention should be: A. Determine when the problematic behavior started. B. Obtain a release to speak to the school about their behavior. C. Explore how each of the family members are adjusting to the new living arrangements. D. Provide psychoeducation about normal adolescent development and behavior.

C. The INITIAL intervention should be for the social worker to explore the effects of the new living arrangements and how each family member is adjusting (C). A, B, and D may be important only after C has been done.

A high school student has been killed in a gang-related shooting. A social worker is assigned to help students deal with the sudden death of their classmate. What should the social worker do FIRST? A. Identify other at-risk students in the school who may be involved in gangs. B. Develop a gang prevention program within the school. C. Provide students with the space to express how they feel. D. Empathize with the student's feelings.

C. The best place to start is to find out how the students are feeling about the incident (C). D would come next. A and B could be done once clients had processed their feelings.

A social worker meets with a new client who describes feelings of anxiety and depression that began soon after she got married. The client's affect is flat and she keeps her eyes averted for much of the session. When asked about recent pleasurable experiences, the client states offhandedly that she is "not allowed to go out much." The client becomes more visibly anxious toward the end of the session and keeps touching a bruise on the side of her face. The social worker should NEXT: A. Encourage the client to attend marital counseling with her partner. B. Encourage the partner to meet with the social worker for an individual session. C. Educate the client about the cycle of violence. D. Refer the client to a psychiatrist.

C. The client is showing signs of DV, so C is the best answer. Once it was ruled out, the social worker could consider A, but only if there was no DV. D would be helpful to do at a later point. There is no reason to do B.

An unlicensed social worker is meeting with an older woman for their first therapy session. The older woman reminds the social worker a lot of her mother who she does not have a good relationship with. The social worker finds herself becoming easily frustrated with the older woman and is having a hard time remaining calm. What should the social worker do first: A. She should bring up the situation with the client and explain why she is becoming agitated. B. Since it is only the first session, she should transfer her to another therapist. C. She should talk to a supervisor about her countertransference issues. D. She should journal about her feelings after the sessions.

C. The experience the social worker is having is a countertransference reaction. When you put the answers in order, the first thing to do would be to talk to a supervisor; this could lead to her doing D or B. A would not be appropriate in the initial phase of treatment, as it makes the social workers issues the focus.

A social worker meets with a client who has a history of DUI and reckless endangerment of his two small children. The client has followed his court-ordered service plan including regular attendance at individual and group treatment sessions, as well as regular drug testing and mandatory employment. In an individual session, he tells his social worker that he wants to see his two children, who are currently in custody with his ex-wife. The social worker should NEXT: A. Speak with the ex-wife regarding the father's wishes. B. Ask the client if the two children want to spend more time with him. C. Address any legal barriers to visitation. D. Schedule a supervised visit for the father.

C. The first thing would be to see if there is any reason he can't see his kids - After that, A, B, or D could be considered.

A social worker is facilitating a group for survivors of childhood sexual abuse. During the most recent session, one of the group members reports drinking excessive amounts of alcohol and taking pain pills to sleep through the night. The social worker should NEXT: A. Provide the group with psychoeducation around substance use and trauma B. Encourage group members to share their reaction about this disclosure C. Meet with the group member individually to discuss a referral for substance use treatment D. Refer the group member for a medical evaluation

C. The social worker should address the group member's substance use needs on an individual basis. A might be a useful intervention but does not address the individual needs of this group member. B may shame the group member by focusing specifically on the behavior and will not get him/her the referral the group member needs for substance use treatment. D may be appropriate at some point but not before the client is referred for substance use treatment.

A social worker has been facilitating a support group for mothers with postpartum depression for nearly a year when she receives a training opportunity that will require her to be gone for several months. The social worker should NEXT: A. Notify the group of her training opportunity and explain why she will be gone B. Meet with each group member individually to process what her absence will mean for them C. Arrange for another social worker to take over the group to ensure the client's needs are met D. Consult with a supervisor about how to terminate with group members

C. The social worker's primary responsibility is to ensure that the client's receive ongoing care and support, making C the best answer. A is not the best answer, because it may go into more detail than is appropriate regarding her absence and does not ensure group members are receiving continued support. B is not the best answer since the social worker should inform the group of her absence as a unit. D does not make sense because it does not address the need for continuity of care

A client tells her social worker in session that she is planning to join a local gardening club. The social worker is the club leader, but the client does not know this. What should the social worker do? A. Step down as club leader. B. Allow the client to join the club but ask her not to disclose to other club members the nature of their relationship. C. Address the conflict with the client. D. Provide the client with other gardening clubs in the area.

C. This is a boundary issue and should be discussed with the client. B and D could be things discussed.

A social worker is meeting with a client involved in a bitter custody dispute with her ex-boyfriend. The client shares that her six-year-old daughter doesn't like her dad's new girlfriend. The client adds, "If I ever hear that his girlfriend does something to my daughter, I'll make sure she pays." What should be the social worker's immediate concern? A. Warn the former girlfriend of the threat. B. Acknowledge the client's feelings about this situation. C. Clarify what the client means by "I'll make sure she pays." D. Focus on the six-year-old daughter's need for peaceful parenting.

C. When a clientmakes a threat of harm (to self or others), the social worker must FIRST clarifythe intent and meaning of the threat. Agoes too far, as the information provided is not sufficient to warrant aTarasoff situation, B and C do not address the potential safety issue.

A white social worker receives a referral from a school counselor for a 5 year-old black male. The counselor reports that the child refuses to stay in his seat, is disruptive to his peers and is oppositional with the teacher. When the social worker meets with the boy's parents, they offer one-word answers to questions and the father sits with his arms folded across his chest. He states, "You people are always targeting us. I bet you don't see any white kids getting referred!" What should the social worker do FIRST? A. Refer the parents to a black social worker. B. Suggest behavioral modification techniques to address the behaviors. C. Clarify the parents' understanding of the reasons for the referral. D. Explore the parents' perceptions of their child's behavior.

C. When clients are referred by a 3rd party and appear hostile or resistant in the initial session, it is important to make sure they know WHY they have been referred. A is premature; B and D may both be appropriate, but not before a relationship is established.

A clinical social worker is evaluating a friendly, 5-year-old child who has difficulty making friends in school. He is very verbal, but has a hard time answering questions about his feelings. He is eager to talk about all the different types of reptiles and knows all about them. What should be the social worker's next step: A. Diagnose him with Autism Spectrum Disorder. B. Use play therapy to try to connect with him on different topics. C. Refer him to a social skills group. D. Suggest that he is transferred to a different school.

C. While there are signs of Austism Spectrum Disorder, there is not enough to make a formal diagnosis. B suggests the social worker direct the play, which you wouldn't do in the initial phase of treatment. The main issue is his inability to make friends and talk about his feelings, so a social skills group is clearly indicated. There is no reason to have him transfer to another school.

A woman comes into session angry because she found out that her boyfriend is cheating on her. She gets enraged and screams 'I'm going to kill him' and runs out. What should the social worker do first? A. Notify the authorities B. Notify the boyfriend C. Call the client D. Wait to talk to her during their next scheduled session

C. You would first want to attempt to contact the client and determine the seriousness of her threat, so C is the best answer. If the client is unavailable or confirms her intention to harm her boyfriend, then the therapist should do A.

A social services agency wants to expand their services to meet the needs of the local Muslim community. The Director of Social Services is assigned the task of identifying the most appropriate services. During the initial phase of planning what is the BEST thing for the director to do? A. Convene a focus group with members of the local community. B. Contact the local mosque and request a meeting. C. Consult with experts in the field of Muslim mental health. D. Survey current Muslim clients to find out what services they need.

C. You would want to consult with experts first, because they would be the best at directing you how to move forward with any of the options presented in A, B or D. The expert could tell you important things like how to word your questions, who to include in the discussion, how to structure focus groups, so you would want to consult with them first to make the most effective plan. This is a challenging question as all answers would be good to do at some point, but getting some expert guidance would be the best place to start.

A social worker at a day treatment facility for teens with mood disorders is asked to provide crisis intervention following the suicide of a program participant. The social worker should FIRST: A. Refrain from sharing information due to confidentiality. B. Contact the residents' parents to inform them of the situation. C. Facilitate a group session to discuss the incident with the other participants. D. Have individual meetings with each participant to discuss the incident.

C. You would want to host a group meeting to ensure that all members hear the same information at the same time so C is the best thing to do first. You could then do B and D. While you would want to be sensitive to issues of confidentiality, you would not have to refrain from sharing any information as it affects the whole community.

A social worker in a community clinic meets with a woman who presents with issues related to her job. She discusses a recent incident in which she was gone from work for several days without notifying her boss or coworkers, although she has no memory of this. The client has found a pamphlet and tickets from a Broadway show in her home, although she states that she's never been to New York. The client is describing which of the following conditions? A. Depersonalization B. Derealization C. Delirium D. Dissociation

D is the best answer, because engaging in activities with no consciousness or memory of the activities is a form of dissociation; A, B and C describe different phenomena.

A client who has recently immigrated to the United States seeks therapy to adjust to the client's culturally different spouse. To assess the client's level of acculturation, which of the following factors should the social worker take into consideration FIRST? A. Newly immigrated individuals are often hesitant to challenge cultural traditions in their marital relationships. B. Newly immigrated individuals are resistant to learning the rules of a new country. C. Gender differences cause more discomfort than acculturation issues. D. The pace of acculturation may vary from group to group.

D is the best answer, because it does not make assumptions and encourages the social worker to understand more about this individual's experience. A and B both make negative assumptions. C is not true.

A social worker at a community agency wants to conduct a needs assessment to determine areas for program development. All of the following activities might be included in a needs assessment EXCEPT: A. Conducting a survey in the community. B. Holding a community forum. C. Collect service statistics regarding use of current programs. D. ID individuals or groups with influence to fund programs.

D is the best answer, because it is unrelated to a needs assessment or the determination of appropriate areas for program development.

The 66-year-old wife of a man with a Mild Neurocognitive Disorder seeks counseling from a social worker due to increasingly stressful caregiving responsibilities. The wife is well-integrated into the mental health and medical care delivery systems since her husband has been receiving a range of treatments since age 55. In order to reduce the wife's sense of burden, the social worker should FIRST help her: A. Provide her with psychoeducation regarding the illness and available treatments. B. Utilize a life review to help her identify her husband's positive contributions. C. Improve her response accuracy to decrease crises. D. Re-evaluate her system of caregiving resources outside the family.

D is the best answer, because it most directly addresses the client's feelings of stress related to caregiving duties. A is probably not necessary since the client has been integrated into delivery systems for over 10 years. B will not address her stress level; C may not be possible given the nature of her husband's illness.

A client diagnosed with an aggressive and painful cancer discloses his wish to end his life because of his terminal illness. How should a social worker with strong beliefs and opinions on this issue manage ethical responsibilities toward this client? A. Refer the client to a social worker who has expertise working with terminal illness. B. Encourage the client to discover a new commitment to living with illness. C. Discuss the different moral values between the social worker and the client. D. Seek consultation to provide treatment within the client's value system.

D is the best answer, because it reflects the ethical values of supporting a client's right to self-determination and providing treatment to clients regardless of their differences from the social worker. A may be necessary if the therapist, with the aid of consultation and other supports, was unable to provide adequate treatment to the client. B and C are both inappropriate.

A client who is gay tells his social worker that he wants to have a baby. He reports that his plan is to contact an adoption agency to begin the process. What should the social worker do FIRST: A. Assess the client's ability to manage the potential negative response from family members. B. Refer the client to a parenting class for single fathers. C. Assess the client's understanding of the steps needed to adopt a child. D. Explore the client's desire to become a parent.

D is the best answer, because it starts where the client is and takes a nonjudgmental stance. A does not make sense, since the stem does not indicate that his family is involved or that they would be unsupportive. B might be helpful later, but not before D. C is somewhat judgmental and assumes that the client has not thought through or researched the decision.

A 53-year-old man comes to a social work agency for help regarding a loss of interest in sexual relations with his wife of 20 years. In the intake interview, the client further describes decreased energy, achiness all over his body and depressed mood. The social worker should FIRST: A. Evaluate the client for sexual dysfunction. B. Assess the client for depression. C. Refer for a psychiatric evaluation. D. Refer for a physical evaluation.

D is the best answer, because the client is describing physical symptoms and depressed mood; the social worker needs to rule out a medical cause for the client's problems before moving forward. A, B, and C could be done after a med issue ruled out.

A social worker begins to notice a pattern of interaction between the client and the social worker. The social worker offers specific suggestions to help the client achieve personal goals and the client thanks the social worker and then identifies the reasons why each suggestion will not work. To promote progress, the social work should FIRST: A. Help the client identify this self-defeating attitude. B. Remind the client of the treatment goals. C. Focus conversations on client's strengths and motivation. D. Point out this pattern to the client.

D is the best option in this case because the client may not be aware of the pattern; simply by sharing the observation of this pattern, the therapist may be able to help the client change. A, B, and C may be viable interventions, but not before the observation of the pattern is shared.

A social worker in private practice has been treating a family for 2 years when the father is unexpectedly laid off from his job. The parents come to the social worker explaining that they can no longer afford to pay the full fee for sessions; the social worker believes the family is in need of further treatment, especially now. What should the social worker do? A. Provide free services to the family. B. Provide the parents with job postings and interview techniques. C. Refer the family to an agency that low-fee community counseling center. D. Provide a sliding scale based fee for the family.

D is the best place to start. A is unethical because it could build up resentment in the therapist. B is not the focus of treatment. C could be done after D and once the family was out of crisis.

A social worker in a community mental health agency meets with a 16-year-old client for the first time. During the session, the social worker asks open-ended questions about the client's history and the presenting problem; the social worker clarifies certain statements but also allows the client to talk uninterrupted. This approach serves all of the following purposes EXCEPT: A. Building therapeutic rapport B. Obtaining diagnostic information C. Assessing for insight D. Developing a treatment plan

D is the correct answer, because the therapist's approach serves the purpose of achieving A, B, and C, but not D.

A company hires a social work consultant after experiencing a noticeable shift in employee morale that the administration believes is impacting productivity. The consultant should FIRST: A. Provide training to administration on ways to improve employee morale B. Identify other reasons for decreased productivity C. Encourage the company to lay off individuals who are having a negative impact on work culture D. Provide space for employees to express their concerns and experiences

D is the firststep in addressing morale issues; employees should have the opportunity toexpress themselves so that targeted solutions can be developed. A and C would be done later; B wouldhopefully be identified as a result of D.

An eight year old boy is brought to therapy by his foster parents. He was removed from his mother's care after it was determined that her boyfriend was physically abusive to him. The social worker offers a game to play but he doesn't respond. The social worker should NEXT: A. Inform the foster parents that he isn't ready for therapy. B. Tell him the game is really fun. C. Talk to him about the abuse he's experienced. D. Invite him to look around the room&point to toy he likes.

D is the intervention most likely to help engage the child and begin building therapeutic rapport; it is not unusual for children to act uninterested when they first arrive in treatment. A may not be true and will not help him. B communicates that the social worker doesn't care if he doesn't want to play the game. C is too direct for an initial session with a child who has experienced trauma.

A bilingual social worker is hired by a charter school to provide clinical services and support to the Spanish-speaking students and families. After a few months, the social worker realizes that the Latino students receive detentions and in-school suspensions more frequently than other students with similar offenses. What should the social worker do FIRST? A. Contact the parents of the suspended Latino students and encourage them to file a complaint. B. Meet with the principal and share your hypothesis. C. Determine which members of the administration target Latino students. D. Document the comparable incidents to support the hypothesis of institutional racism.

D would be the best place to start. Once you have data, it may tell you C or you could meet with the principal (B). You should wait to do A until further steps have been taken.

A social worker is facilitating a group for single adults with depression. During a session, one group member shares that he recently started dating a woman but is unsure if he should keep seeing her. He reports that the woman has taken him shopping, changed his wardrobe, and has insisted that he cut his hair and start reading certain books. What should the social worker do NEXT: A. Meet with the client individually to discuss this relationship B. Explore why the client is allowing his new girlfriend to exert so much influence on him C. Encourage group members to share their reactions to his new relationship D. Encourage the client to identify how he wants to respond to his girlfriend

D. A is not the best answer, since the client is bringing the issue up in group and there is no reason to separate him to address this. B makes a judgment about the client's choices and behavior. While it may be helpful to have the group share their reactions to his relationship (C), the NEXT thing you would want to do is allow the client an opportunity to identify how he wants to respond to his girlfriend.

A 13-year-old girl meets with a social worker after being discharged from a psychiatric hospital. During the assessment, the social worker learns that the child has a history of suicidal ideation and aggressive behavior at school and home. These behaviors have become worse over the past year. The family also has a history of mental illness, and the client's father is in jail. After conducting a risk assessment, the social worker should NEXT: A. Explore the child's aggressive behaviors. B. Encourage the mother to place her daughter in a residential treatment home. C. Evaluate the child's behaviors that occur at school. D. Evaluate the overall functioning of the family and the child.

D. After risk, the next step would be to assess the family. This would determine the need for B. A could be done later.

A social worker meets with a 22-year-old client who recently immigrated from Iran to attend college. The client is uncomfortable in social situations and is having difficulty finding friends. In one session, the social worker reflects the client's stated feelings about disconnection and missing home. The client becomes offended at the social worker's comments and says, "You have no idea what I'm talking about." The social worker should NEXT: A. Restate the reflection so the client feels understood. B. Explore any cultural issues that are coming up. C. Refer the client to a Persian social worker. D. Encourage the client to further clarify his statement.

D. Asking the client what s/he means is the best answer. If the client mentions a cultural issue, then B would be next. You cannot do A until you understand what the client means.

A community mental health agency recently hired several new social work graduates for their outpatient treatment program. The social work supervisor is facilitating an orientation for the new hires, which will be MOST effective if the supervisor: A. Provides the new hires with a manual of policies and procedure B. Has several staff members present on their own experiences as employees C. Encourages each new hire to present on their areas of specialization D. Gives an overview of the agency's mission and values

D. Helping new staff understand the overall goals and mission of the agency is the best way to orient them to the workplace. A, B, and C are all reasonable activities but will not be more effective that D.

A 30-year-old man meets with a social worker for the first time and describes himself as a failure. He recently lost his job and his girlfriend of two years broke up with him. He feels hopeless about his life and is beginning to believe his family's criticism and rejection of him are justified. As the interview progresses, his discouragement and feelings of worthlessness become more apparent. The social worker should FIRST: A. Refer the client to a psychiatrist for a medication evaluation. B. Help the client identify why his relationship with his girlfriend ended. C. Schedule a family session. D. Ask the client if he's had any thoughts about hurting himself.

D. Hopelessness and feeling like a failure are risk signs for suicide. Once D was ruled out, A, B, and C could be considered.

A social worker meets with a young adult who is considering looking for her biological parents. The client struggles with the fear that her adoptive parents would reject her if she reunites with her birth parents. What should the social worker do FIRST? A. Explore resources for adopted adults with the client. B. Facilitate a family session to help the client disclose the plan. C. Assist the client in identifying ways to talk about this plan within the family. D. Clarify the client's own issues and feelings related to being adopted.

D. It is important to start where the client is. D is an effort to find out how the client feels about the situation. A, B, and C could be done later.

A social worker in community agency has been working with a client with Borderline Personality Disorder for 2 years; the social worker has begun to feel that the client needs involvement in a Dialectical Behavioral Treatment (DBT) program in order to improve. The client is hospitalized for a suicide attempt before the social worker can discuss this treatment option. The social worker should: A. Suggest that the hospital refer client to local DBT program. B. Contact the DBT program & refer client upon discharge. C. Call client in the hospital to discuss termination and referral. D. Resume work with the client after discharge.

D. It would be inappropriate for the social worker to attempt to terminate and refer the client when the client is in the midst of a crisis, making A, B and C all poor choices. In order to avoid client abandonment, the therapist should resume work with the client after discharge to assure that the client is stable and would then begin discussing termination and referral.

A school social worker has received a referral for a 6-year-old boy who is having behavior issues in the class. As part of the assessment, the social worker asks the parents to sign a consent for a classroom observation, but the parents refuse, stating they are uncomfortable with this. What should the social worker do NEXT: A. ​​​​​​Point out the benefits the observation could have on the situation B. Inquire about the parents' resistance to the observation C. Facilitate a meeting with the parents and school administration regarding the boy's behavior D. Acknowledge the parents' right to make the decision

D. Reinforcing the parents' right to make the decision supports their self-determination and the relationship. A could be construed as coercive and could damage the relationship. B and C may both be helpful, but not before the social worker supports their self-determination.

When discussing the fee with a potential new client the social worker learns that the client recently lost her job and is living off her savings. Even after sliding the fee to the lowest amount, the client states that she only thinks she can afford four or five sessions. The social worker should: A. Agree to see her at the reduced rate. B. Lower the rate to accommodate her situation. C. Find out what issues the client hopes to deal with in therapy. D. Refer the client to a low-fee mental health services.

D. Since no relationship has been established with the client yet, the social worker should provide the client with referrals to a low-fee mental health agency (D). A and B would be inaccurate because four or five sessions at a reduced rate would most likely not be effective for the client. C is inaccurate because it doesn't address the client's inability to pay for therapy.

A hospital-based social worker receives outcome data that indicates a large percentage of geriatric patients with knee replacements are functioning at lower levels one year after surgery. As part of the medical team's response, the social worker is asked to develop a psychosocial program that will enhance functioning. The social worker's BEST approach is to: A. Provide pre-operation workshops to highlight challenges. B. Coordinate the group with the medical director. C. Recommend additional physical rehabilitation. D. Create a drop-in support group.

D. Since the question asks about a psychosocial program, C does not make sense, since it refers to physical rehab. A does not address the patients' functioning after surgery and B is an incomplete answer.

A young couple tells their social worker that they are afraid to enroll their 5 year-old in Kindergarten and that they most likely will not do so, but plan to homeschool the child instead. What should the social worker do FIRST? A. Encourage the parents to visit local schools. B. Provide the parents with psychoeducation about the benefits of Kindergarten. C. Explore the parents' own schooling experience. D. Explore the parents' feelings about sending their child to school.

D. Start where the clients are and explore their own feelings about sending their child to school. A, B, and C can be done later.

A social worker at a women's shelter is asked to see a woman in crisis who has been abused by her husband. The woman enters the social worker's office stating that she plans to return home in the morning and "just needed to spend the night somewhere else and talk to someone about this." The MOST appropriate intervention the social worker can make is to: A. Explore the woman's reasons for wanting to return home. B. Encourage the woman to address her feelings about her husband. C. Provide psychoeducation on the cycle of violence. D. Develop a safety plan with the woman.

D. The client has already made a decision. Her safety is the most important issue to address. A is trying to change her mind. B and C would be fine to do, but D is the most important.

A 69-year-old diabetic client is referred by her doctor. She recently had her leg amputated because she hasn't been following her insulin and diet plan. The client cries in session and tells you "I just can't manage all this and I don't see it getting better." What should the social worker do FIRST? A. Strengthen the client's social support network. B. Get a release to consult with the doctor about her treatment. C. Develop a plan for medical compliance. D. Assess client for potential suicide.

D. The client'sstated feelings of hopelessness make D the best place to start. A, B, and C would all be done after ensuringthe client's safety.

A mother seeks treatment for her 16-year-old daughter. The mother is concerned that until now the daughter was a good student and enjoyed involvement with their family, church and school. They rarely disagreed. For the past three months, however, the daughter has wanted to spend all her time with her friends. The daughter has kept her grades up, but arguments have occurred around when she has to spend time with the family and how often she can talk on the phone with her friends. What should the social worker do FIRST? A. Suggest family therapy to find compromises. B. Support the mother's efforts to maintain structure and discipline. C. Refer the daughter for individual therapy. D. Provide the parents with education about normal adolescent behavior.

D. The daughter's behavior demonstrates typical adolescent development and the parents may be more successful in intervening with her if they understand this; A may not be necessary if the parents' understand the daughter's behavior better; B will probably only serve to cause more conflict; C is inappropriate since the presenting problem is a family dynamic.

A non-profit agency in a low-income housing development receives a grant to provide housing residents with opportunities to become involved in local and state politics. The goal is to affect policy and improve socioeconomic conditions. What approach is the social worker MOST likely using? A. Ecological Model B. Program development C. Peer Training D. Empowerment

D. The grant isspecifically to provide opportunities to involve housing residents in local andstate politics, which would best be achieved through the Empowerment Model.When engaging in community social work, empowerment of clients is a keycomponent of our work.

A school social worker meets with an 18-year-old high school senior whose teacher referred them. The teacher reported the client seemed distracted in school, was turning in assignments late, and had a noticeable drop in grades despite having previously been a good student. When meeting with the student, she reports that she doesn't want to be seen for counseling and says that, "only crazies go to therapy." She says she just wants to get out of school and is ready to go away to college. What should the social worker do FIRST? A. Explore the client's strong desire to leave school and go to college B. Explore any psychosocial stressors that are occuring in school or at home C. Complete a comprehensive assessment D. Explore the client's negative connotations around being in therapy

D. The therapist needs to first explore why the client is ambivalent to attend therapy and what is significant about the meaning of therapy. Once that has been addressed the therapist may then be able to explore A and B as part of the comprehensive assessment, C.

A social work intern is presenting a case to a multidisciplinary diagnostic team for review. During the presentation, the social worker identifies social, cultural, physiological, economic, and environmental factors that interconnect and impact her client's current problem. The intern is using: A. Biological Perspective B. Ethical Perspective C. Policy Perspective D. Ecological Perspective

D. This is a factual recall question. A, B, and C would all be described differently.

A social worker has been working with a client who abuses alcohol. After a month, the client begins to acknowledge that his drinking is having a negative impact on his life and wants to make his life better. During the preparation stage of change, all of the following are possible interventions EXCEPT: A. Clarify client's goals and strategies. B. Mobilize support systems. C. Discuss treatment options. D. Explore family history of abuse.

D. This is a factual recall question; A, B, and C are all possible interventions during the preparation stage of change.

A social worker is meeting with a pregnant client who shares that she plans to have an abortion because the father of the baby is a married man who does not want to the baby. The social worker disagrees with the client's plan to have an abortion, but does not express her disagreement and instead showed active listening and interest. What quality is the social worker exhibiting? A. Sublimation B. Acceptance C. Reaction formation D. Unconditional positive regard

D. This is a perfect example of unconditional positive regard. What's being described is not the social worker merely accepting her decision (B), but choosing to show unconditional positive regard despite the disagreement. (A) and (C) are defense mechanisms that operate outside of awareness.

A social worker facilitates an interdisciplinary team that is holding an emergency meeting to discuss a 27-year-old man with an intellectual disability. The majority of the team feels that the client is in need of some type of psychotropic medication because of seriously escalating aggressive and impulsive behaviors. The physician member of the team refuses to consider that suggestion and states that team members are "outside of their scope of practice." The social worker's BEST course of action is: A. Remind team members that decisions involving medication must be made by a physician. B. Arrange to meet with the physician individually to address his concerns. C. Reexamine the need for psychotropic medications. D. Refocus the team's discussion on the client's behaviors.

D. When a multidisciplinary team disagrees about how to intervene, refocusing the conversation on the client's behaviors and progress is often helpful. A may be true but will not help the team move forward. B would potentially damage the team's cohesiveness and ability to work together. C does not make sense, since the stem clearly tells us that the team disagrees about medication.

A school social worker has been assigned to work with an 8-year-old girl referred by her teacher for profanity with peers. What should the social worker to FIRST? A. Schedule a family conference. B. Request a medical evaluation. C. Report child abuse. D. Confer with the teacher about the behavior.

D. When school social workers receive referrals, the FIRST step is to obtain information from the referral source (usually the teacher); A and B may be appropriate steps later in the process. C does not make sense, since there is no indication of child abuse in the stem.

An 8-year old child with developmental delays is currently in a regular classroom setting. He is struggling to keep up with the work and he is noticing he is not being assigned the same work material as his classmates. The social worker should first: A. Advocate for him to receive an aid in the classroom. B. Transfer him to a special education classroom. C. Assess him using the mental status exam. D. Speak to his classmates about his situation.

The best answer is A as you would do this before having him transferred to a special education class. There is no reason to do a mental status exam at this point. D would be breaking the child's confidentiality.

While working with a 14-year-old girl in a group foster home, she tells the social worker that she reminds her of her mother. The girl had a long history of abuse and neglect by the mother. What is the best way for the social worker to deal with this transference issue? A. Use the transference to assist the client in working through unresolved issues with her mother. B. Immediately assign her to a different social worker. C. Seek consultation to avoid boundary issues. D. Talk to the staff at the foster home to make sure everyone is on the same page.

The best answer is A. The social worker can use the transference therapeutically. There is no reason to transfer the client, seek consultation, or talk to the staff at the at this point

A woman is in school to be a nurse and is currently studying about stomach cancer. She starts getting severe pains in her stomach and goes to the ER who medically clears her. She keeps experiencing the same pains and sees 5 other specialists who all medically clear her. She goes to a social worker and complains that the Doctors aren't 'doing their job'. What should be the first intervention the social worker should take: A. Empathize how frustrating it must feel. B. Confront her about her denial. C. Help her to find another Doctor who will be more sympathetic to her ailments. D. Provide psychoeducation about what she is experiencing.

The best answer is A. You would want to empathize with her feelings. You could go on to do D. B would be dismissing the client's experience. And you would not be expected to do C.

A mother brings her adolescent son to therapy following the recent death of his grandmother. During the initial session, the boy tells the social worker "I was close to my grandmother, but I am doing fine. I don't know why my mother brought me here." What is the BEST course of action for the social worker to take? A. Ask the boy about his relationship with his mother. B. Ask the boy about his relationship with grandmother. C. Conduct a psychosocial assessment. D. Refer the boy to a grief support group.

The best answer is C. It would include doing both A and B. There are times on the exam where one answer will actually include the other good options. That is the case here. Also, C would include finding out additional information. You would not want to do D until you had completed the assessment.

In establishing a therapeutic alliance with a client from a different racial or ethnic background, the FIRST issue for the social worker to address is: A. The racial or ethnic difference and how it may impact treatment. B. Transference issues with the therapist. C. The therapist's countertransference. D. The client's history of encounters with people of different racial or ethnic backgrounds.

The best answer option is A, because it deals directly with the client-therapist relationship and brings the topic up for exploration. B might be done later in treatment but not FIRST. C would be addressed in consultation or supervision if the therapist felt it was interfering with treatment but would not be done FIRST. D would not do anything to help establish a therapeutic alliance between the client and social worker.

A 72-year-old woman has been in treatment with a social worker for 6 months for depression and anxiety. One day, she arrives thirty minutes late for a regularly scheduled appointment. She reports that she got confused on the bus and couldn't remember which stop to get off at. The social worker should FIRST: A. Call the client's family and inform them of the incident. B. Call the client a taxi for the ride home. C. Encourage the client to get a physical examination. D. Recommend immediate hospitalization for further evaluation.

The best answeris C, because it addresses the concern without taking away the client'sself-determination.

A school principal told parents of an autism spectrum child that the school does not have special services needed for their child. What is the school social worker's BEST action? A. Advocate for more services B. Find another school which has special services C. Assist fam in calling board of education to file a complaint D. Recommend home school for the child

The best course of action would be to advocate for more services so A is the best option. B and C could be done if advocating is ineffective. There is no reason to suggest D.

A 10-year-old boy is placed with a foster family that has different religious beliefs than the child. The foster parents want the foster child to follow their religious practices. What is the social worker's BEST course of action in this situation? A. Tell the family to follow the child's religion. B. Instruct the child to follow the family's religion. C. Tell the child not to practice any religion to avoid conflict. D. Identify alternate placement for the child.

The best option here is D. All the other options go against honoring the diversity of the people. Some people may not pick this because they think that another option would not be available, but if it is an option on the exam then it is available.

A couple starts seeing a social worker after the death of their two children in a car accident. The social worker's supervisor notices that the social worker is spending many hours on the phone with the couple and is seeing them more often than her other clients. The supervisor should: A. Work with the social worker to assess if the level of intervention is necessary. B. Advise the social worker that her behavior is unethical. C. Transfer the clients immediately to a different social worker. D. Have the social worker tell the clients that they can't contact her after hours.

The best placeto start would be A. This is an exampleof the supervisor recognizing that the social worker's boundaries may becompromised, so the best response would be to discuss it with the socialworker.


Ensembles d'études connexes

Assignment 5 & 7 & 8 & 9 & 10-Media & Society

View Set

Name of This Book Is Secret, The by Pseudonymous Bosch

View Set

Nclex Style: Nursing Fundamentals - Fluids and Electrolytes

View Set

Chapter 3 Globalization and sport

View Set